Đến nội dung

cristianoronaldo

cristianoronaldo

Đăng ký: 02-05-2016
Offline Đăng nhập: 19-04-2022 - 10:51
****-

#691302 Tìm GTNN:$P=\left ( 2-xyz \right )\left ( \sum\...

Gửi bởi cristianoronaldo trong 22-08-2017 - 20:25

$\boxed{\text{Bài toán}}$(cristianoronaldo)

Cho x,y,z là các số thực dương thỏa mãn $xy+yz+zx=3$.

Tìm GTNN của biểu thức:

$P=\left ( 2-xyz \right )\left ( \frac{1}{x\sqrt{y^2+z^2}}+\frac{1}{y\sqrt{z^2+x^2}}+\frac{1}{z\sqrt{x^2+y^2}} \right )$




#691283 $\boxed{\text{TOPIC}}$ Ôn thi học si...

Gửi bởi cristianoronaldo trong 22-08-2017 - 17:20

Một bài ở mức higher level nhỉ :)

Bài toán 6:(VMO) Cho các số thực dương $x_{1}; x_{2};...:x_{n}$ thỏa mãn: $\sum^{n}_{i=1} \frac{1}{1+x_{i}}=1$

Hãy chứng minh rằng:

$\prod^{n}_{i=1}x_{i}\geq (n-1)^n$ 

Theo bài ra ta có:

$\sum_{i=1}^{n} \frac{1}{1+x_{i}}=1\Leftrightarrow \sum _{i=1}^{n-1}\frac{1}{1+x_{i}}=\frac{x_{n}}{1+x_{n}}$

Áp dụng bất đẳng thức AM-GM ta có:

$\frac{x_{n}}{1+x_{n}}=\sum _{i=1}^{n-1}\frac{1}{1+x_{i}}\geq \frac{n-1}{\sqrt[n-1]{\prod _{i=1}^{n-1}(1+x_{i})}}$

Tương tự $n-1$ bất đẳng thức còn lại, nhân vế ta được Q.E.D




#691282 $\boxed{\text{TOPIC}}$ Ôn thi học si...

Gửi bởi cristianoronaldo trong 22-08-2017 - 17:10

$\boxed{\text{Bài 7}}$:

Cho x,y,z là các số thực không âm thỏa mãn $y\geq x> z> 0$.

Tìm GTNN của biểu thức:

$\sqrt{xy}+\frac{z(x-y)}{\sqrt{xy}}+\frac{1}{z(x-z)}$

$\boxed{\text{Bài 8}}$(cristianoronaldo)

Cho x,y,z là các số thực dương thỏa mãn $xy+yz+zx=3$.

Tìm GTNN của biểu thức:

$P=\left ( 2-xyz \right )\left ( \frac{1}{x\sqrt{y^2+z^2}}+\frac{1}{y\sqrt{z^2+x^2}}+\frac{1}{z\sqrt{x^2+y^2}} \right )$




#690955 $\sum \sqrt{1+\frac{48x}{y+z}}\geq 15$ .

Gửi bởi cristianoronaldo trong 18-08-2017 - 21:31

Bài toán 1 : Cho $x,y,z \in$ R . Chứng minh với mọi tam giác ABC ta có :

 $(\frac{b}{c}+\frac{c}{b}-1)x^2+(\frac{c}{a}+\frac{a}{c}-1)y^2+(\frac{a}{b}+\frac{b}{a}-1)z^2$

                                              $\geq (3-\frac{b+c}{a})yz+(3-\frac{c+a}{b})zx+(3-\frac{a+b}{c})xy$

( Note : a,b,c là 3 cạnh của tam giác ABC )

Bài toán 2 : Cho $x,y,z \geq 0$ và 2 trong chúng không đồng thời bằng 0 . Chứng minh rằng :

                  $\sqrt{1+\frac{48x}{y+z}}+\sqrt{1+\frac{48y}{z+x}}+\sqrt{1+\frac{48z}{x+y}}\geq 15$ .

Lần này anh chắc chắn đúng rồi

*Bổ đề: Cho a,b,c là các số thực không âm thỏa mãn $c=min\left \{ a,b,c \right \}$ thì ta có:

$\sqrt{\frac{a}{b+c}}+\sqrt{\frac{b}{c+a}}\geq 2\sqrt{\frac{a+b}{a+b+2c}}$

Chứng minh:

Áp dụng bất đẳng thức Holder ta có:

$\left ( \sqrt{\frac{a}{b+c}}+\sqrt{\frac{b}{c+a}} \right )^2\left [ a^2(b+c)+b^2(c+a) \right ]\geq (a+b)^3$

$\Rightarrow \sqrt{\frac{a}{b+c}}+\sqrt{\frac{b}{c+a}}\geq \sqrt{\frac{(a+b)^3}{a^2(b+c)+b^2(c+a)}}$

Ta thấy:

$a^2(b+c)+b^2(c+a)=ab(a+b-2c)+c(a+b)^2$

$\leq \left ( \frac{a+b}{2} \right )^2(a+b-2c)+c(a+b)^2$

$=\frac{(a+b)^3+2c(a+b)^2}{4}$

Như vậy ta có:

$\sqrt{\frac{a}{b+c}}+\sqrt{\frac{b}{c+a}}\geq \sqrt{\frac{4(a+b)^3}{(a+b)^3+2c(a+b)^2}}=2\sqrt{\frac{a+b}{a+b+2c}}$         

Bổ đề được chứng minh.

Bây giờ quay trở lại bài toán.

Không mất tính tổng quát ta giả sử $z=min\left \{ x,y,z \right \}$.

Áp dụng bất đẳng thức Minkowski ta có:

$\sqrt{1+\frac{48x}{y+z}}+\sqrt{1+\frac{48y}{z+x}}\geq \sqrt{4+48\left ( \sqrt{\frac{x}{y+z}}+\sqrt{\frac{y}{z+x}} \right )^2}$

 $\geq \sqrt{4+48\left ( 2\sqrt{\frac{x+y}{x+y+2z}} \right )^2}=2\sqrt{1+\frac{48(x+y)}{x+y+2z}}$

Như vậy ta cần chứng minh:

$2\sqrt{1+\frac{48(x+y)}{x+y+2z}}+\sqrt{1+\frac{48z}{x+y}}\geq 15$

Đặt $\sqrt{1+\frac{48z}{x+y}}=t \left ( 1\leq t\leq 5 \right )$

Đến đây đưa về bất đẳng thức về một biến là xong.

$\Rightarrow Q.E.D$                        




#690695 Cmr:$\frac{x+y}{z}+\frac{y+z}...

Gửi bởi cristianoronaldo trong 16-08-2017 - 21:01

$\boxed{\text{Bài toán}}$:

Cho x,y,z là các số thực dương thỏa mãn $xyz=1$. Chứng minh rằng:

$\frac{x+y}{z}+\frac{y+z}{x}+\frac{z+x}{y}\geq \sqrt{3\left ( x^3+y^3+z^3 \right )}+3$

 

                                                                                        $\boxed{\text{cristianoronaldo}}$




#690625 $\sum \sqrt[3]{\frac{x+y}{2z}...

Gửi bởi cristianoronaldo trong 15-08-2017 - 22:23

(Đề chọn ĐT Chuyên Phan Bội Châu 2014)

Cho $x,y,z$ là các số thực dương thay đổi sao cho $xyz=1$. Chứng minh rằng:

$\sum \sqrt[3]{\frac{x+y}{2z}}  \leq \frac{5(x+y+z)+9}{8}$

Đặt $x=a^3,y=b^3,z=c^3$ thì $abc=1$ và bất đẳng thức cần chứng minh tương đương với:

$\sum ab\sqrt[3]{\frac{a^3+b^3}{2}}\leq \frac{5(\sum a^3 )+9}{8}$

Áp dụng bất đẳng thức AM-GM ta có:

$ab\sqrt[3]{\frac{a^3+b^3}{2}}=\sqrt[3]{\frac{a+b}{2}.ab.ab.ab.(a^2-ab+b^2)}$

                       $\leq \sqrt[3]{\frac{a+b}{2}.\left ( \frac{3ab+a^2-ab+b^2}{4} \right )^4}$

                       $=\left ( \frac{a+b}{2} \right )^3$

Như vậy ta cần phải chứng minh:

$\sum \left ( \frac{a+b}{2} \right )^3\leq \frac{5(\sum a^3)+9}{8}=\frac{5(\sum a^3)+9abc}{8}$

Khai triển được bất đẳng thức Schur bậc 3.

$\Rightarrow Q.E.D$




#690600 Chứng minh BĐT

Gửi bởi cristianoronaldo trong 15-08-2017 - 18:41

Cho x, y không âm thỏa mãn $x^{2}+y^{2}=1$. Chứng minh rằng 

$xy+ max(x;y) \leq \frac{3\sqrt{3}}{4}$

Không mất tính tổng quát, giả sử $x=max\left \{ x,y \right \}$

Áp dụng bất đẳng thức AM-GM ta có:

$xy+max\left \{ x,y \right \}=xy+x\leq \frac{x^2+3y^2}{2\sqrt{3}}+\frac{4x^2+3}{4\sqrt{3}}=\frac{6(x^2+y^2)+3}{4\sqrt{3}}=\frac{3\sqrt{3}}{4}$

$\Rightarrow Q.E.D$




#690598 Cho a,b,c>0 và abc=1 CMR: P=$\sum 1/(a^2+ab) >= 3/2$

Gửi bởi cristianoronaldo trong 15-08-2017 - 18:30

Cho  a,b,c>0 và abc=1  CMR: P=$\sum 1/(a^2+ab) >= 3/2$

Do $abc=1$ nên tồn tại số thực dương x,y,z thỏa mãn:$a=\frac{x}{y},b=\frac{z}{x},c=\frac{y}{z}$

Khi đó bất đẳng thức trở thành:

$P=\sum \frac{y^2}{x^2+yz}\geq \frac{3}{2}$

Áp dụng bất đẳng thức Cauchy-SchwarzVasc ta có:

$P=\sum \frac{y^4}{x^2y^2+y^3z}\geq \frac{(\sum x^2)^2}{\sum x^2y^2+\sum x^3y}\geq \frac{(\sum x^2)^2}{\frac{1}{3}(\sum x^2)^2+\frac{1}{3}(\sum x^2)^2}=\frac{3}{2}$

$\Rightarrow Q.E.D$




#690594 Min $\frac{a}{b}+2\sqrt{1+\frac...

Gửi bởi cristianoronaldo trong 15-08-2017 - 17:57

Áp dụng bất đẳng thức AM-GM ta có:

$P\geq \frac{a}{b}+2\sqrt{2\sqrt{\frac{b}{c}}}+3\sqrt[3]{2\sqrt{\frac{c}{a}}}=\frac{a}{b}+2\sqrt{2}\sqrt[4]{\frac{b}{c}}+3\sqrt[3]{2}\sqrt[6]{\frac{c}{a}}$

    $=\frac{\sqrt{2}}{2}\left ( \frac{a}{b}+4\sqrt[4]{\frac{b}{c}}+6\sqrt[6]{\frac{c}{a}} \right )+\frac{2-\sqrt{2}}{2}.\frac{a}{b}-3\left ( \sqrt{2}-\sqrt[3]{2} \right )\sqrt[6]{\frac{c}{a}}$

    $\geq \frac{\sqrt{2}}{2}.11\sqrt[11]{\frac{a}{b}\left ( \sqrt[4]{\frac{b}{c}}\right )^4\left ( \sqrt[6]{\frac{c}{a}} \right )^6}+1-\frac{\sqrt{2}}{2}-3(\sqrt{2}-\sqrt[3]{2})$

    $=1+2\sqrt{2}+3\sqrt[3]{2}$

$\Rightarrow Q.E.D$




#689676 Cmr:$6+27(\sum ab)\leq 5\left [\sum \frac{...

Gửi bởi cristianoronaldo trong 05-08-2017 - 20:55

$\boxed{\text{Bài toán}}$:

Cho a,b,c là các số thực thỏa mãn $a+b+c=1$. Chứng minh rằng:

$6+27(ab+bc+ca)\leq 5\left [ \frac{a(3-4b^2-4c^2)}{a^2+1}+\frac{b(3-4c^2-4a^2)}{b^2+1}+\frac{c(3-4a^2-b^2)}{c^2+1} \right ]$

 

                                                                                                       $\boxed{\text{cristianoronaldo}}$




#689658 Tìm GTLN:P=$\sum\frac{3x-2\sqrt{(1-y)(1-z)...

Gửi bởi cristianoronaldo trong 05-08-2017 - 19:48

$\boxed{\text{Bài toán}}$:

Cho x,y,z là các số thực dương thỏa mãn:$\left\{\begin{matrix} min\left \{ \sqrt{x}+\sqrt{y},\sqrt{y}+\sqrt{z},\sqrt{z}+\sqrt{x} \right \}> \sqrt{5}-1\\ x+y+z=2\end{matrix}\right.$.

Tìm GTLN của biểu thức:

$P=\frac{3x-2\sqrt{(1-y)(1-z)}}{y+z}+\frac{3y-2\sqrt{(1-z)(1-x)}}{z+x}+\frac{3z-2\sqrt{(1-x)(1-y)}}{x+y}$

 

                                                                                                                           $\boxed{\text{cristianoronaldo}}$ 




#688939 $\sum \frac{a}{bc\sqrt{b^2+c^2}} \geq \frac{abc...

Gửi bởi cristianoronaldo trong 28-07-2017 - 20:06

$\boxed{\text{Bài toán}}$:

Cho a,b,c là các số thực dương thỏa mãn $a+b+c=1$. Chứng minh rằng:

$\frac{a}{bc\sqrt{b^2+c^2}}+\frac{b}{ca\sqrt{c^2+a^2}}+\frac{c}{ab\sqrt{a^2+b^2}}\geq \frac{abc\sqrt{2}+2ab^{3}+2bc^{3}+2ca^{3}}{2abc\left [ (\sqrt{2}-1)abc+2ab^{3}+2bc^{3}+2ca^{3} \right ]}$

 

                                                                                                                  $\boxed{\text{cristianoronaldo}}$

 

 

Spoiler




#688850 Cmr:$\frac{a}{b+c}+\frac{b}...

Gửi bởi cristianoronaldo trong 27-07-2017 - 20:37

$\boxed{\text{Bài toán}}$:

Cho a,b,c là các số thực dương thỏa mãn $a\geq b+c$. Chứng minh rằng:

$\frac{a}{b+c}+\frac{b}{c+a}+\frac{c}{a+b}+\sqrt{\frac{abc}{(a+b)(b+c)(c+a)}}\geq 2$

 

 

 

 




#688732 Chứng minh: $\sum \frac{X_{n}}{\...

Gửi bởi cristianoronaldo trong 26-07-2017 - 17:55

cho n số dương x1 ,...,xcó tổng bằng 1 .CMR 

 

$\frac{X_{1}}{\sqrt{1-X_{1}}} +... +\frac{X_{n}}{\sqrt{1-X_{n}}} \geq$ $\frac{\sqrt{x_1}+...+\sqrt{x_n}}{\sqrt{n-1}}$

Áp dụng bất đẳng thức Cauchy-Schwarz ta có:

$\sum \frac{x_{1}}{\sqrt{1-x_{1}}}\geq \frac{(\sum x_{1})^2}{\sum \sqrt{x_{1}}.\sqrt{x_{1}-x_{1}^{2}}}\geq \frac{1}{\sqrt{\sum x_{1}}.\sqrt{\sum (x_{1}-x_{1}^{2})}}\geq \sqrt{\frac{n}{n-1}}$

Mặt khác:

$\frac{\sum \sqrt{x_{1}}}{\sqrt{n-1}}\leq \sqrt{\frac{n}{n-1}}$

$\Rightarrow Q.E.D$




#688586 CMR:$\frac{1}{a^2+a+1}+\frac{a^2...

Gửi bởi cristianoronaldo trong 25-07-2017 - 08:22

Cho $a,b$ là các số thực dương. Chứng minh rằng: $\frac{1}{a^2+a+1}+\frac{a^2}{a^2+ab+b^2}+\frac{b^2}{b^2+b+1}\ge 1$

Đặt $\left\{\begin{matrix} a=x\\ \frac{b}{a}=y\\ \frac{1}{b}=z\end{matrix}\right.$$(x,y,z> 0)$ thì $xyz=1$

Bất đẳng thức tương đương vs:

$\sum \frac{1}{x^2+x+1}\geq 1$

Đây là bất đẳng thức Vasc

$\Rightarrow Q.E.D$